Proof the Legendre polynomial $P_n$ has $n$ distinct real zeros












6












$begingroup$


I need a proof to show that the inequality $m < n$ leads to a contradiction and $P_n$ has $n$ distinct real roots, all of which lie in the open interval $(-1, 1)$.










share|cite|improve this question











$endgroup$












  • $begingroup$
    What is the meaning of $m$?
    $endgroup$
    – gammatester
    Oct 15 '13 at 14:02


















6












$begingroup$


I need a proof to show that the inequality $m < n$ leads to a contradiction and $P_n$ has $n$ distinct real roots, all of which lie in the open interval $(-1, 1)$.










share|cite|improve this question











$endgroup$












  • $begingroup$
    What is the meaning of $m$?
    $endgroup$
    – gammatester
    Oct 15 '13 at 14:02
















6












6








6


6



$begingroup$


I need a proof to show that the inequality $m < n$ leads to a contradiction and $P_n$ has $n$ distinct real roots, all of which lie in the open interval $(-1, 1)$.










share|cite|improve this question











$endgroup$




I need a proof to show that the inequality $m < n$ leads to a contradiction and $P_n$ has $n$ distinct real roots, all of which lie in the open interval $(-1, 1)$.







polynomials roots orthogonal-polynomials






share|cite|improve this question















share|cite|improve this question













share|cite|improve this question




share|cite|improve this question








edited Mar 24 '14 at 12:50









Antonio Vargas

20.7k245111




20.7k245111










asked Oct 15 '13 at 13:53









richitesenpairichitesenpai

1,1451027




1,1451027












  • $begingroup$
    What is the meaning of $m$?
    $endgroup$
    – gammatester
    Oct 15 '13 at 14:02




















  • $begingroup$
    What is the meaning of $m$?
    $endgroup$
    – gammatester
    Oct 15 '13 at 14:02


















$begingroup$
What is the meaning of $m$?
$endgroup$
– gammatester
Oct 15 '13 at 14:02






$begingroup$
What is the meaning of $m$?
$endgroup$
– gammatester
Oct 15 '13 at 14:02












3 Answers
3






active

oldest

votes


















5












$begingroup$

By Rodrigues formula for Legendre polynomials,
$$displaystyle P_n(x) = frac{1}{2^nn!}frac{d^n}{dx^n} (x^2 - 1)^{n}tag{*1}$$
$P_n(x)$ is the $n^{th}$ derivative of a polynomial with roots at $-1$ and $1$.
Repeat applying Gauss-Lucas theorem $n$ times, we know all roots of $P_n(x)$ lie on the closed line segment
$[-1,1]$ in the complex plane. By direct expansion of $(*1)$, one can check that
$P_n(pm 1) ne 0$, so the roots of $P_n(x)$ are all real and belongs to $(-1,1)$.



Since Legendre polynomials are solutions of the Legendre's differential equation:
$$frac{d}{dx}left[(1-x^2)frac{d}{dx}P_n(x)right] + n(n+1)P_n(x) = 0$$
which is a $2^{nd}$ order ODE, $P_n(x)$ cannot have any double roots. To see this,
let's say $P_n(x)$ has a double root at $alpha in (-1,1)$.
$P_n(x)$ will then be a solution of following initial value problem:



$$frac{d}{dx}left[(1-x^2)frac{d}{dx}y(x)right] + n(n+1)y(x) = 0,quad
begin{cases}y(alpha) = 0,\y'(alpha) = 0end{cases}$$
Apply Picard-Lindelöf theorem to this $2^{nd}$ order ODE, the "uniqueness" part of the theorem tell us $P_n(x)$ vanishes over some neighbor of $alpha$. Since $P_n(x)$ is not identically zero, this is impossible. As a consequence, all roots of $P_n(x)$ are simple.






share|cite|improve this answer











$endgroup$













  • $begingroup$
    @Sam: If you think the author of the Accepted Answer made a substantive error, you should point out your concern in a Comment rather than making a unilateral change by Editing.
    $endgroup$
    – hardmath
    Apr 22 '18 at 22:31





















12












$begingroup$

From the wording of your question I guess you are looking for something like this:



The Legendre polynomial $P_n$ with $n>0$ has $n$ simple roots in $(-1, 1).$



Proof by contradiction: Assume $P_n$ has $m$ with $0 le m lt n;$
pairwise different zeroes $x_1, x_2, dots x_m$ of odd multiplicity in $(-1,1),;$ i.e. $P_n$ changes sign in $x_i$, and consider the polynomial $Z_n(x) = (x-x_1)cdot (x-x_2) dots (x-x_m).;$ Because $P_n$ is orthogonal to $P_0$ the intgeral $int_{-1}^{1} P_n(x) dx$ vanishes and $P_n$ has at least one zero of odd multiplicity in $(-1,1),,$ i.e. $m>0.;$



The polynomial $Z_n(x) P_n(x)$ does not change sign in $(-1,1)$ and therefore $I_n = int_{-1}^{1} Z_n(x) P_n(x) dx ne 0.;$ On the other hand $Z_n$ has degree $m$ and can be written as a linear combination $Z_n = sum_{k=0}^m c_k P_k(x)$. Thus we get the contradiction by means of the orthogonality of the $P_k$:
begin{align}
0 ne I_n &= int_{-1}^{1} Z_n(x) P_n(x) dx\
&= int_{-1}^{1} P_n(x) sum_{k=0}^m c_k P_k(x) dx\
&= sum_{k=0}^m c_k int_{-1}^{1} P_n(x) P_k(x) dx\
&= 0
end{align}



This shows that $P_n$ has $n$ pairwise different zeroes of odd multiplicity, and because $P_n$ has at most $n$ zeroes, the zeroes $x_1,dots, x_n$ are simple.



Note that this kind of proof can be applied to general orthogonal polynomials over intervals $(a,b)$ with weighting function $w(x) ge 0$.






share|cite|improve this answer











$endgroup$













  • $begingroup$
    I don't see why "The polynomial $Z_n(x) P_n(x)$ does not change sign in $(-1,1)$". Suppose for example $P_n(x)$ has a root of multiplicity 3 in $(-1,1)$. Then $Z_n(x)P_n(x)$ will change sign at that point.
    $endgroup$
    – 6005
    Dec 18 '13 at 14:32










  • $begingroup$
    @Goos: A root of multiplicity 3 is not simple.
    $endgroup$
    – gammatester
    Dec 18 '13 at 15:19






  • 1




    $begingroup$
    I am aware of that. Let $a$ be the root of multiplicity $3$. $Z_n(x)$ will not change sign at $a$ (since $a$ is not simple), and $P_n(x)$ will change sign (since $a$ has multiplicity 3). Hence $Z_n(x)P_n(x)$ will change sign.
    $endgroup$
    – 6005
    Dec 18 '13 at 15:29










  • $begingroup$
    @Goos: I see what you mean and I refined the proof by showing that $P_n$ has $n$ roots of odd multiplicity.
    $endgroup$
    – gammatester
    Dec 19 '13 at 7:55








  • 1




    $begingroup$
    Nice approach. But the proof only works if you have shown that there is at least one odd zero? Otherwise $Z_n = 0$ and there is no contradiction.
    $endgroup$
    – andreas
    Mar 24 '14 at 12:07



















0












$begingroup$

Let Legendre polynomial be $$ f_n(x)=frac{1}{2^n n!}frac{d^{n}}{d x^n}(x^2-1)^n .$$



Note that $ (x^2-1)^n $ has roots $ -1, 1 $ each with multiplicity $ n $, and $$ (x^2-1)Big|frac{d^m}{d x^{m}}(x^2-1)^n, , text{when} m<n .$$



Thus, $ -1, 1 $ are always roots of $ frac{d^{m}}{d x^m}(x^2-1)^n $. And by Rolle's theorem, we know that there are $ n-1 $ distinct zeros $ x_1, x_2, cdots, x_{n-1} $ in the interval $ (-1, 1) $ for the polynomial $ frac{d^{n-1}}{d x^{n-1}}(x^2-1)^n $. Since $ -1, 1 $ are also zeros of $ frac{d ^{n-1}}{d x^{n-1}}(x^2-1)^n $. Use Rolle's theorem once again, we get $ n $ distinct zeros in $ (-1, 1) $ and since $ deg (frac{d^{n}}{d x^{n}}(x^2-1)^n)=n $, we have found all the distinct roots of $ f_n(x) $ in $ (-1, 1) $.






share|cite|improve this answer









$endgroup$













    Your Answer





    StackExchange.ifUsing("editor", function () {
    return StackExchange.using("mathjaxEditing", function () {
    StackExchange.MarkdownEditor.creationCallbacks.add(function (editor, postfix) {
    StackExchange.mathjaxEditing.prepareWmdForMathJax(editor, postfix, [["$", "$"], ["\\(","\\)"]]);
    });
    });
    }, "mathjax-editing");

    StackExchange.ready(function() {
    var channelOptions = {
    tags: "".split(" "),
    id: "69"
    };
    initTagRenderer("".split(" "), "".split(" "), channelOptions);

    StackExchange.using("externalEditor", function() {
    // Have to fire editor after snippets, if snippets enabled
    if (StackExchange.settings.snippets.snippetsEnabled) {
    StackExchange.using("snippets", function() {
    createEditor();
    });
    }
    else {
    createEditor();
    }
    });

    function createEditor() {
    StackExchange.prepareEditor({
    heartbeatType: 'answer',
    autoActivateHeartbeat: false,
    convertImagesToLinks: true,
    noModals: true,
    showLowRepImageUploadWarning: true,
    reputationToPostImages: 10,
    bindNavPrevention: true,
    postfix: "",
    imageUploader: {
    brandingHtml: "Powered by u003ca class="icon-imgur-white" href="https://imgur.com/"u003eu003c/au003e",
    contentPolicyHtml: "User contributions licensed under u003ca href="https://creativecommons.org/licenses/by-sa/3.0/"u003ecc by-sa 3.0 with attribution requiredu003c/au003e u003ca href="https://stackoverflow.com/legal/content-policy"u003e(content policy)u003c/au003e",
    allowUrls: true
    },
    noCode: true, onDemand: true,
    discardSelector: ".discard-answer"
    ,immediatelyShowMarkdownHelp:true
    });


    }
    });














    draft saved

    draft discarded


















    StackExchange.ready(
    function () {
    StackExchange.openid.initPostLogin('.new-post-login', 'https%3a%2f%2fmath.stackexchange.com%2fquestions%2f527140%2fproof-the-legendre-polynomial-p-n-has-n-distinct-real-zeros%23new-answer', 'question_page');
    }
    );

    Post as a guest















    Required, but never shown

























    3 Answers
    3






    active

    oldest

    votes








    3 Answers
    3






    active

    oldest

    votes









    active

    oldest

    votes






    active

    oldest

    votes









    5












    $begingroup$

    By Rodrigues formula for Legendre polynomials,
    $$displaystyle P_n(x) = frac{1}{2^nn!}frac{d^n}{dx^n} (x^2 - 1)^{n}tag{*1}$$
    $P_n(x)$ is the $n^{th}$ derivative of a polynomial with roots at $-1$ and $1$.
    Repeat applying Gauss-Lucas theorem $n$ times, we know all roots of $P_n(x)$ lie on the closed line segment
    $[-1,1]$ in the complex plane. By direct expansion of $(*1)$, one can check that
    $P_n(pm 1) ne 0$, so the roots of $P_n(x)$ are all real and belongs to $(-1,1)$.



    Since Legendre polynomials are solutions of the Legendre's differential equation:
    $$frac{d}{dx}left[(1-x^2)frac{d}{dx}P_n(x)right] + n(n+1)P_n(x) = 0$$
    which is a $2^{nd}$ order ODE, $P_n(x)$ cannot have any double roots. To see this,
    let's say $P_n(x)$ has a double root at $alpha in (-1,1)$.
    $P_n(x)$ will then be a solution of following initial value problem:



    $$frac{d}{dx}left[(1-x^2)frac{d}{dx}y(x)right] + n(n+1)y(x) = 0,quad
    begin{cases}y(alpha) = 0,\y'(alpha) = 0end{cases}$$
    Apply Picard-Lindelöf theorem to this $2^{nd}$ order ODE, the "uniqueness" part of the theorem tell us $P_n(x)$ vanishes over some neighbor of $alpha$. Since $P_n(x)$ is not identically zero, this is impossible. As a consequence, all roots of $P_n(x)$ are simple.






    share|cite|improve this answer











    $endgroup$













    • $begingroup$
      @Sam: If you think the author of the Accepted Answer made a substantive error, you should point out your concern in a Comment rather than making a unilateral change by Editing.
      $endgroup$
      – hardmath
      Apr 22 '18 at 22:31


















    5












    $begingroup$

    By Rodrigues formula for Legendre polynomials,
    $$displaystyle P_n(x) = frac{1}{2^nn!}frac{d^n}{dx^n} (x^2 - 1)^{n}tag{*1}$$
    $P_n(x)$ is the $n^{th}$ derivative of a polynomial with roots at $-1$ and $1$.
    Repeat applying Gauss-Lucas theorem $n$ times, we know all roots of $P_n(x)$ lie on the closed line segment
    $[-1,1]$ in the complex plane. By direct expansion of $(*1)$, one can check that
    $P_n(pm 1) ne 0$, so the roots of $P_n(x)$ are all real and belongs to $(-1,1)$.



    Since Legendre polynomials are solutions of the Legendre's differential equation:
    $$frac{d}{dx}left[(1-x^2)frac{d}{dx}P_n(x)right] + n(n+1)P_n(x) = 0$$
    which is a $2^{nd}$ order ODE, $P_n(x)$ cannot have any double roots. To see this,
    let's say $P_n(x)$ has a double root at $alpha in (-1,1)$.
    $P_n(x)$ will then be a solution of following initial value problem:



    $$frac{d}{dx}left[(1-x^2)frac{d}{dx}y(x)right] + n(n+1)y(x) = 0,quad
    begin{cases}y(alpha) = 0,\y'(alpha) = 0end{cases}$$
    Apply Picard-Lindelöf theorem to this $2^{nd}$ order ODE, the "uniqueness" part of the theorem tell us $P_n(x)$ vanishes over some neighbor of $alpha$. Since $P_n(x)$ is not identically zero, this is impossible. As a consequence, all roots of $P_n(x)$ are simple.






    share|cite|improve this answer











    $endgroup$













    • $begingroup$
      @Sam: If you think the author of the Accepted Answer made a substantive error, you should point out your concern in a Comment rather than making a unilateral change by Editing.
      $endgroup$
      – hardmath
      Apr 22 '18 at 22:31
















    5












    5








    5





    $begingroup$

    By Rodrigues formula for Legendre polynomials,
    $$displaystyle P_n(x) = frac{1}{2^nn!}frac{d^n}{dx^n} (x^2 - 1)^{n}tag{*1}$$
    $P_n(x)$ is the $n^{th}$ derivative of a polynomial with roots at $-1$ and $1$.
    Repeat applying Gauss-Lucas theorem $n$ times, we know all roots of $P_n(x)$ lie on the closed line segment
    $[-1,1]$ in the complex plane. By direct expansion of $(*1)$, one can check that
    $P_n(pm 1) ne 0$, so the roots of $P_n(x)$ are all real and belongs to $(-1,1)$.



    Since Legendre polynomials are solutions of the Legendre's differential equation:
    $$frac{d}{dx}left[(1-x^2)frac{d}{dx}P_n(x)right] + n(n+1)P_n(x) = 0$$
    which is a $2^{nd}$ order ODE, $P_n(x)$ cannot have any double roots. To see this,
    let's say $P_n(x)$ has a double root at $alpha in (-1,1)$.
    $P_n(x)$ will then be a solution of following initial value problem:



    $$frac{d}{dx}left[(1-x^2)frac{d}{dx}y(x)right] + n(n+1)y(x) = 0,quad
    begin{cases}y(alpha) = 0,\y'(alpha) = 0end{cases}$$
    Apply Picard-Lindelöf theorem to this $2^{nd}$ order ODE, the "uniqueness" part of the theorem tell us $P_n(x)$ vanishes over some neighbor of $alpha$. Since $P_n(x)$ is not identically zero, this is impossible. As a consequence, all roots of $P_n(x)$ are simple.






    share|cite|improve this answer











    $endgroup$



    By Rodrigues formula for Legendre polynomials,
    $$displaystyle P_n(x) = frac{1}{2^nn!}frac{d^n}{dx^n} (x^2 - 1)^{n}tag{*1}$$
    $P_n(x)$ is the $n^{th}$ derivative of a polynomial with roots at $-1$ and $1$.
    Repeat applying Gauss-Lucas theorem $n$ times, we know all roots of $P_n(x)$ lie on the closed line segment
    $[-1,1]$ in the complex plane. By direct expansion of $(*1)$, one can check that
    $P_n(pm 1) ne 0$, so the roots of $P_n(x)$ are all real and belongs to $(-1,1)$.



    Since Legendre polynomials are solutions of the Legendre's differential equation:
    $$frac{d}{dx}left[(1-x^2)frac{d}{dx}P_n(x)right] + n(n+1)P_n(x) = 0$$
    which is a $2^{nd}$ order ODE, $P_n(x)$ cannot have any double roots. To see this,
    let's say $P_n(x)$ has a double root at $alpha in (-1,1)$.
    $P_n(x)$ will then be a solution of following initial value problem:



    $$frac{d}{dx}left[(1-x^2)frac{d}{dx}y(x)right] + n(n+1)y(x) = 0,quad
    begin{cases}y(alpha) = 0,\y'(alpha) = 0end{cases}$$
    Apply Picard-Lindelöf theorem to this $2^{nd}$ order ODE, the "uniqueness" part of the theorem tell us $P_n(x)$ vanishes over some neighbor of $alpha$. Since $P_n(x)$ is not identically zero, this is impossible. As a consequence, all roots of $P_n(x)$ are simple.







    share|cite|improve this answer














    share|cite|improve this answer



    share|cite|improve this answer








    edited Apr 23 '18 at 3:26

























    answered Oct 15 '13 at 14:38









    achille huiachille hui

    95.8k5132258




    95.8k5132258












    • $begingroup$
      @Sam: If you think the author of the Accepted Answer made a substantive error, you should point out your concern in a Comment rather than making a unilateral change by Editing.
      $endgroup$
      – hardmath
      Apr 22 '18 at 22:31




















    • $begingroup$
      @Sam: If you think the author of the Accepted Answer made a substantive error, you should point out your concern in a Comment rather than making a unilateral change by Editing.
      $endgroup$
      – hardmath
      Apr 22 '18 at 22:31


















    $begingroup$
    @Sam: If you think the author of the Accepted Answer made a substantive error, you should point out your concern in a Comment rather than making a unilateral change by Editing.
    $endgroup$
    – hardmath
    Apr 22 '18 at 22:31






    $begingroup$
    @Sam: If you think the author of the Accepted Answer made a substantive error, you should point out your concern in a Comment rather than making a unilateral change by Editing.
    $endgroup$
    – hardmath
    Apr 22 '18 at 22:31













    12












    $begingroup$

    From the wording of your question I guess you are looking for something like this:



    The Legendre polynomial $P_n$ with $n>0$ has $n$ simple roots in $(-1, 1).$



    Proof by contradiction: Assume $P_n$ has $m$ with $0 le m lt n;$
    pairwise different zeroes $x_1, x_2, dots x_m$ of odd multiplicity in $(-1,1),;$ i.e. $P_n$ changes sign in $x_i$, and consider the polynomial $Z_n(x) = (x-x_1)cdot (x-x_2) dots (x-x_m).;$ Because $P_n$ is orthogonal to $P_0$ the intgeral $int_{-1}^{1} P_n(x) dx$ vanishes and $P_n$ has at least one zero of odd multiplicity in $(-1,1),,$ i.e. $m>0.;$



    The polynomial $Z_n(x) P_n(x)$ does not change sign in $(-1,1)$ and therefore $I_n = int_{-1}^{1} Z_n(x) P_n(x) dx ne 0.;$ On the other hand $Z_n$ has degree $m$ and can be written as a linear combination $Z_n = sum_{k=0}^m c_k P_k(x)$. Thus we get the contradiction by means of the orthogonality of the $P_k$:
    begin{align}
    0 ne I_n &= int_{-1}^{1} Z_n(x) P_n(x) dx\
    &= int_{-1}^{1} P_n(x) sum_{k=0}^m c_k P_k(x) dx\
    &= sum_{k=0}^m c_k int_{-1}^{1} P_n(x) P_k(x) dx\
    &= 0
    end{align}



    This shows that $P_n$ has $n$ pairwise different zeroes of odd multiplicity, and because $P_n$ has at most $n$ zeroes, the zeroes $x_1,dots, x_n$ are simple.



    Note that this kind of proof can be applied to general orthogonal polynomials over intervals $(a,b)$ with weighting function $w(x) ge 0$.






    share|cite|improve this answer











    $endgroup$













    • $begingroup$
      I don't see why "The polynomial $Z_n(x) P_n(x)$ does not change sign in $(-1,1)$". Suppose for example $P_n(x)$ has a root of multiplicity 3 in $(-1,1)$. Then $Z_n(x)P_n(x)$ will change sign at that point.
      $endgroup$
      – 6005
      Dec 18 '13 at 14:32










    • $begingroup$
      @Goos: A root of multiplicity 3 is not simple.
      $endgroup$
      – gammatester
      Dec 18 '13 at 15:19






    • 1




      $begingroup$
      I am aware of that. Let $a$ be the root of multiplicity $3$. $Z_n(x)$ will not change sign at $a$ (since $a$ is not simple), and $P_n(x)$ will change sign (since $a$ has multiplicity 3). Hence $Z_n(x)P_n(x)$ will change sign.
      $endgroup$
      – 6005
      Dec 18 '13 at 15:29










    • $begingroup$
      @Goos: I see what you mean and I refined the proof by showing that $P_n$ has $n$ roots of odd multiplicity.
      $endgroup$
      – gammatester
      Dec 19 '13 at 7:55








    • 1




      $begingroup$
      Nice approach. But the proof only works if you have shown that there is at least one odd zero? Otherwise $Z_n = 0$ and there is no contradiction.
      $endgroup$
      – andreas
      Mar 24 '14 at 12:07
















    12












    $begingroup$

    From the wording of your question I guess you are looking for something like this:



    The Legendre polynomial $P_n$ with $n>0$ has $n$ simple roots in $(-1, 1).$



    Proof by contradiction: Assume $P_n$ has $m$ with $0 le m lt n;$
    pairwise different zeroes $x_1, x_2, dots x_m$ of odd multiplicity in $(-1,1),;$ i.e. $P_n$ changes sign in $x_i$, and consider the polynomial $Z_n(x) = (x-x_1)cdot (x-x_2) dots (x-x_m).;$ Because $P_n$ is orthogonal to $P_0$ the intgeral $int_{-1}^{1} P_n(x) dx$ vanishes and $P_n$ has at least one zero of odd multiplicity in $(-1,1),,$ i.e. $m>0.;$



    The polynomial $Z_n(x) P_n(x)$ does not change sign in $(-1,1)$ and therefore $I_n = int_{-1}^{1} Z_n(x) P_n(x) dx ne 0.;$ On the other hand $Z_n$ has degree $m$ and can be written as a linear combination $Z_n = sum_{k=0}^m c_k P_k(x)$. Thus we get the contradiction by means of the orthogonality of the $P_k$:
    begin{align}
    0 ne I_n &= int_{-1}^{1} Z_n(x) P_n(x) dx\
    &= int_{-1}^{1} P_n(x) sum_{k=0}^m c_k P_k(x) dx\
    &= sum_{k=0}^m c_k int_{-1}^{1} P_n(x) P_k(x) dx\
    &= 0
    end{align}



    This shows that $P_n$ has $n$ pairwise different zeroes of odd multiplicity, and because $P_n$ has at most $n$ zeroes, the zeroes $x_1,dots, x_n$ are simple.



    Note that this kind of proof can be applied to general orthogonal polynomials over intervals $(a,b)$ with weighting function $w(x) ge 0$.






    share|cite|improve this answer











    $endgroup$













    • $begingroup$
      I don't see why "The polynomial $Z_n(x) P_n(x)$ does not change sign in $(-1,1)$". Suppose for example $P_n(x)$ has a root of multiplicity 3 in $(-1,1)$. Then $Z_n(x)P_n(x)$ will change sign at that point.
      $endgroup$
      – 6005
      Dec 18 '13 at 14:32










    • $begingroup$
      @Goos: A root of multiplicity 3 is not simple.
      $endgroup$
      – gammatester
      Dec 18 '13 at 15:19






    • 1




      $begingroup$
      I am aware of that. Let $a$ be the root of multiplicity $3$. $Z_n(x)$ will not change sign at $a$ (since $a$ is not simple), and $P_n(x)$ will change sign (since $a$ has multiplicity 3). Hence $Z_n(x)P_n(x)$ will change sign.
      $endgroup$
      – 6005
      Dec 18 '13 at 15:29










    • $begingroup$
      @Goos: I see what you mean and I refined the proof by showing that $P_n$ has $n$ roots of odd multiplicity.
      $endgroup$
      – gammatester
      Dec 19 '13 at 7:55








    • 1




      $begingroup$
      Nice approach. But the proof only works if you have shown that there is at least one odd zero? Otherwise $Z_n = 0$ and there is no contradiction.
      $endgroup$
      – andreas
      Mar 24 '14 at 12:07














    12












    12








    12





    $begingroup$

    From the wording of your question I guess you are looking for something like this:



    The Legendre polynomial $P_n$ with $n>0$ has $n$ simple roots in $(-1, 1).$



    Proof by contradiction: Assume $P_n$ has $m$ with $0 le m lt n;$
    pairwise different zeroes $x_1, x_2, dots x_m$ of odd multiplicity in $(-1,1),;$ i.e. $P_n$ changes sign in $x_i$, and consider the polynomial $Z_n(x) = (x-x_1)cdot (x-x_2) dots (x-x_m).;$ Because $P_n$ is orthogonal to $P_0$ the intgeral $int_{-1}^{1} P_n(x) dx$ vanishes and $P_n$ has at least one zero of odd multiplicity in $(-1,1),,$ i.e. $m>0.;$



    The polynomial $Z_n(x) P_n(x)$ does not change sign in $(-1,1)$ and therefore $I_n = int_{-1}^{1} Z_n(x) P_n(x) dx ne 0.;$ On the other hand $Z_n$ has degree $m$ and can be written as a linear combination $Z_n = sum_{k=0}^m c_k P_k(x)$. Thus we get the contradiction by means of the orthogonality of the $P_k$:
    begin{align}
    0 ne I_n &= int_{-1}^{1} Z_n(x) P_n(x) dx\
    &= int_{-1}^{1} P_n(x) sum_{k=0}^m c_k P_k(x) dx\
    &= sum_{k=0}^m c_k int_{-1}^{1} P_n(x) P_k(x) dx\
    &= 0
    end{align}



    This shows that $P_n$ has $n$ pairwise different zeroes of odd multiplicity, and because $P_n$ has at most $n$ zeroes, the zeroes $x_1,dots, x_n$ are simple.



    Note that this kind of proof can be applied to general orthogonal polynomials over intervals $(a,b)$ with weighting function $w(x) ge 0$.






    share|cite|improve this answer











    $endgroup$



    From the wording of your question I guess you are looking for something like this:



    The Legendre polynomial $P_n$ with $n>0$ has $n$ simple roots in $(-1, 1).$



    Proof by contradiction: Assume $P_n$ has $m$ with $0 le m lt n;$
    pairwise different zeroes $x_1, x_2, dots x_m$ of odd multiplicity in $(-1,1),;$ i.e. $P_n$ changes sign in $x_i$, and consider the polynomial $Z_n(x) = (x-x_1)cdot (x-x_2) dots (x-x_m).;$ Because $P_n$ is orthogonal to $P_0$ the intgeral $int_{-1}^{1} P_n(x) dx$ vanishes and $P_n$ has at least one zero of odd multiplicity in $(-1,1),,$ i.e. $m>0.;$



    The polynomial $Z_n(x) P_n(x)$ does not change sign in $(-1,1)$ and therefore $I_n = int_{-1}^{1} Z_n(x) P_n(x) dx ne 0.;$ On the other hand $Z_n$ has degree $m$ and can be written as a linear combination $Z_n = sum_{k=0}^m c_k P_k(x)$. Thus we get the contradiction by means of the orthogonality of the $P_k$:
    begin{align}
    0 ne I_n &= int_{-1}^{1} Z_n(x) P_n(x) dx\
    &= int_{-1}^{1} P_n(x) sum_{k=0}^m c_k P_k(x) dx\
    &= sum_{k=0}^m c_k int_{-1}^{1} P_n(x) P_k(x) dx\
    &= 0
    end{align}



    This shows that $P_n$ has $n$ pairwise different zeroes of odd multiplicity, and because $P_n$ has at most $n$ zeroes, the zeroes $x_1,dots, x_n$ are simple.



    Note that this kind of proof can be applied to general orthogonal polynomials over intervals $(a,b)$ with weighting function $w(x) ge 0$.







    share|cite|improve this answer














    share|cite|improve this answer



    share|cite|improve this answer








    edited Mar 24 '14 at 12:46

























    answered Oct 16 '13 at 6:43









    gammatestergammatester

    16.7k21632




    16.7k21632












    • $begingroup$
      I don't see why "The polynomial $Z_n(x) P_n(x)$ does not change sign in $(-1,1)$". Suppose for example $P_n(x)$ has a root of multiplicity 3 in $(-1,1)$. Then $Z_n(x)P_n(x)$ will change sign at that point.
      $endgroup$
      – 6005
      Dec 18 '13 at 14:32










    • $begingroup$
      @Goos: A root of multiplicity 3 is not simple.
      $endgroup$
      – gammatester
      Dec 18 '13 at 15:19






    • 1




      $begingroup$
      I am aware of that. Let $a$ be the root of multiplicity $3$. $Z_n(x)$ will not change sign at $a$ (since $a$ is not simple), and $P_n(x)$ will change sign (since $a$ has multiplicity 3). Hence $Z_n(x)P_n(x)$ will change sign.
      $endgroup$
      – 6005
      Dec 18 '13 at 15:29










    • $begingroup$
      @Goos: I see what you mean and I refined the proof by showing that $P_n$ has $n$ roots of odd multiplicity.
      $endgroup$
      – gammatester
      Dec 19 '13 at 7:55








    • 1




      $begingroup$
      Nice approach. But the proof only works if you have shown that there is at least one odd zero? Otherwise $Z_n = 0$ and there is no contradiction.
      $endgroup$
      – andreas
      Mar 24 '14 at 12:07


















    • $begingroup$
      I don't see why "The polynomial $Z_n(x) P_n(x)$ does not change sign in $(-1,1)$". Suppose for example $P_n(x)$ has a root of multiplicity 3 in $(-1,1)$. Then $Z_n(x)P_n(x)$ will change sign at that point.
      $endgroup$
      – 6005
      Dec 18 '13 at 14:32










    • $begingroup$
      @Goos: A root of multiplicity 3 is not simple.
      $endgroup$
      – gammatester
      Dec 18 '13 at 15:19






    • 1




      $begingroup$
      I am aware of that. Let $a$ be the root of multiplicity $3$. $Z_n(x)$ will not change sign at $a$ (since $a$ is not simple), and $P_n(x)$ will change sign (since $a$ has multiplicity 3). Hence $Z_n(x)P_n(x)$ will change sign.
      $endgroup$
      – 6005
      Dec 18 '13 at 15:29










    • $begingroup$
      @Goos: I see what you mean and I refined the proof by showing that $P_n$ has $n$ roots of odd multiplicity.
      $endgroup$
      – gammatester
      Dec 19 '13 at 7:55








    • 1




      $begingroup$
      Nice approach. But the proof only works if you have shown that there is at least one odd zero? Otherwise $Z_n = 0$ and there is no contradiction.
      $endgroup$
      – andreas
      Mar 24 '14 at 12:07
















    $begingroup$
    I don't see why "The polynomial $Z_n(x) P_n(x)$ does not change sign in $(-1,1)$". Suppose for example $P_n(x)$ has a root of multiplicity 3 in $(-1,1)$. Then $Z_n(x)P_n(x)$ will change sign at that point.
    $endgroup$
    – 6005
    Dec 18 '13 at 14:32




    $begingroup$
    I don't see why "The polynomial $Z_n(x) P_n(x)$ does not change sign in $(-1,1)$". Suppose for example $P_n(x)$ has a root of multiplicity 3 in $(-1,1)$. Then $Z_n(x)P_n(x)$ will change sign at that point.
    $endgroup$
    – 6005
    Dec 18 '13 at 14:32












    $begingroup$
    @Goos: A root of multiplicity 3 is not simple.
    $endgroup$
    – gammatester
    Dec 18 '13 at 15:19




    $begingroup$
    @Goos: A root of multiplicity 3 is not simple.
    $endgroup$
    – gammatester
    Dec 18 '13 at 15:19




    1




    1




    $begingroup$
    I am aware of that. Let $a$ be the root of multiplicity $3$. $Z_n(x)$ will not change sign at $a$ (since $a$ is not simple), and $P_n(x)$ will change sign (since $a$ has multiplicity 3). Hence $Z_n(x)P_n(x)$ will change sign.
    $endgroup$
    – 6005
    Dec 18 '13 at 15:29




    $begingroup$
    I am aware of that. Let $a$ be the root of multiplicity $3$. $Z_n(x)$ will not change sign at $a$ (since $a$ is not simple), and $P_n(x)$ will change sign (since $a$ has multiplicity 3). Hence $Z_n(x)P_n(x)$ will change sign.
    $endgroup$
    – 6005
    Dec 18 '13 at 15:29












    $begingroup$
    @Goos: I see what you mean and I refined the proof by showing that $P_n$ has $n$ roots of odd multiplicity.
    $endgroup$
    – gammatester
    Dec 19 '13 at 7:55






    $begingroup$
    @Goos: I see what you mean and I refined the proof by showing that $P_n$ has $n$ roots of odd multiplicity.
    $endgroup$
    – gammatester
    Dec 19 '13 at 7:55






    1




    1




    $begingroup$
    Nice approach. But the proof only works if you have shown that there is at least one odd zero? Otherwise $Z_n = 0$ and there is no contradiction.
    $endgroup$
    – andreas
    Mar 24 '14 at 12:07




    $begingroup$
    Nice approach. But the proof only works if you have shown that there is at least one odd zero? Otherwise $Z_n = 0$ and there is no contradiction.
    $endgroup$
    – andreas
    Mar 24 '14 at 12:07











    0












    $begingroup$

    Let Legendre polynomial be $$ f_n(x)=frac{1}{2^n n!}frac{d^{n}}{d x^n}(x^2-1)^n .$$



    Note that $ (x^2-1)^n $ has roots $ -1, 1 $ each with multiplicity $ n $, and $$ (x^2-1)Big|frac{d^m}{d x^{m}}(x^2-1)^n, , text{when} m<n .$$



    Thus, $ -1, 1 $ are always roots of $ frac{d^{m}}{d x^m}(x^2-1)^n $. And by Rolle's theorem, we know that there are $ n-1 $ distinct zeros $ x_1, x_2, cdots, x_{n-1} $ in the interval $ (-1, 1) $ for the polynomial $ frac{d^{n-1}}{d x^{n-1}}(x^2-1)^n $. Since $ -1, 1 $ are also zeros of $ frac{d ^{n-1}}{d x^{n-1}}(x^2-1)^n $. Use Rolle's theorem once again, we get $ n $ distinct zeros in $ (-1, 1) $ and since $ deg (frac{d^{n}}{d x^{n}}(x^2-1)^n)=n $, we have found all the distinct roots of $ f_n(x) $ in $ (-1, 1) $.






    share|cite|improve this answer









    $endgroup$


















      0












      $begingroup$

      Let Legendre polynomial be $$ f_n(x)=frac{1}{2^n n!}frac{d^{n}}{d x^n}(x^2-1)^n .$$



      Note that $ (x^2-1)^n $ has roots $ -1, 1 $ each with multiplicity $ n $, and $$ (x^2-1)Big|frac{d^m}{d x^{m}}(x^2-1)^n, , text{when} m<n .$$



      Thus, $ -1, 1 $ are always roots of $ frac{d^{m}}{d x^m}(x^2-1)^n $. And by Rolle's theorem, we know that there are $ n-1 $ distinct zeros $ x_1, x_2, cdots, x_{n-1} $ in the interval $ (-1, 1) $ for the polynomial $ frac{d^{n-1}}{d x^{n-1}}(x^2-1)^n $. Since $ -1, 1 $ are also zeros of $ frac{d ^{n-1}}{d x^{n-1}}(x^2-1)^n $. Use Rolle's theorem once again, we get $ n $ distinct zeros in $ (-1, 1) $ and since $ deg (frac{d^{n}}{d x^{n}}(x^2-1)^n)=n $, we have found all the distinct roots of $ f_n(x) $ in $ (-1, 1) $.






      share|cite|improve this answer









      $endgroup$
















        0












        0








        0





        $begingroup$

        Let Legendre polynomial be $$ f_n(x)=frac{1}{2^n n!}frac{d^{n}}{d x^n}(x^2-1)^n .$$



        Note that $ (x^2-1)^n $ has roots $ -1, 1 $ each with multiplicity $ n $, and $$ (x^2-1)Big|frac{d^m}{d x^{m}}(x^2-1)^n, , text{when} m<n .$$



        Thus, $ -1, 1 $ are always roots of $ frac{d^{m}}{d x^m}(x^2-1)^n $. And by Rolle's theorem, we know that there are $ n-1 $ distinct zeros $ x_1, x_2, cdots, x_{n-1} $ in the interval $ (-1, 1) $ for the polynomial $ frac{d^{n-1}}{d x^{n-1}}(x^2-1)^n $. Since $ -1, 1 $ are also zeros of $ frac{d ^{n-1}}{d x^{n-1}}(x^2-1)^n $. Use Rolle's theorem once again, we get $ n $ distinct zeros in $ (-1, 1) $ and since $ deg (frac{d^{n}}{d x^{n}}(x^2-1)^n)=n $, we have found all the distinct roots of $ f_n(x) $ in $ (-1, 1) $.






        share|cite|improve this answer









        $endgroup$



        Let Legendre polynomial be $$ f_n(x)=frac{1}{2^n n!}frac{d^{n}}{d x^n}(x^2-1)^n .$$



        Note that $ (x^2-1)^n $ has roots $ -1, 1 $ each with multiplicity $ n $, and $$ (x^2-1)Big|frac{d^m}{d x^{m}}(x^2-1)^n, , text{when} m<n .$$



        Thus, $ -1, 1 $ are always roots of $ frac{d^{m}}{d x^m}(x^2-1)^n $. And by Rolle's theorem, we know that there are $ n-1 $ distinct zeros $ x_1, x_2, cdots, x_{n-1} $ in the interval $ (-1, 1) $ for the polynomial $ frac{d^{n-1}}{d x^{n-1}}(x^2-1)^n $. Since $ -1, 1 $ are also zeros of $ frac{d ^{n-1}}{d x^{n-1}}(x^2-1)^n $. Use Rolle's theorem once again, we get $ n $ distinct zeros in $ (-1, 1) $ and since $ deg (frac{d^{n}}{d x^{n}}(x^2-1)^n)=n $, we have found all the distinct roots of $ f_n(x) $ in $ (-1, 1) $.







        share|cite|improve this answer












        share|cite|improve this answer



        share|cite|improve this answer










        answered Dec 6 '18 at 14:37









        user549397user549397

        1,4061417




        1,4061417






























            draft saved

            draft discarded




















































            Thanks for contributing an answer to Mathematics Stack Exchange!


            • Please be sure to answer the question. Provide details and share your research!

            But avoid



            • Asking for help, clarification, or responding to other answers.

            • Making statements based on opinion; back them up with references or personal experience.


            Use MathJax to format equations. MathJax reference.


            To learn more, see our tips on writing great answers.




            draft saved


            draft discarded














            StackExchange.ready(
            function () {
            StackExchange.openid.initPostLogin('.new-post-login', 'https%3a%2f%2fmath.stackexchange.com%2fquestions%2f527140%2fproof-the-legendre-polynomial-p-n-has-n-distinct-real-zeros%23new-answer', 'question_page');
            }
            );

            Post as a guest















            Required, but never shown





















































            Required, but never shown














            Required, but never shown












            Required, but never shown







            Required, but never shown

































            Required, but never shown














            Required, but never shown












            Required, but never shown







            Required, but never shown







            Popular posts from this blog

            Quarter-circle Tiles

            build a pushdown automaton that recognizes the reverse language of a given pushdown automaton?

            Mont Emei